Mo Logo [Home] [Lexikon] [Aufgaben] [Tests] [Kurse] [Begleitmaterial] [Hinweise] [Mitwirkende] [Publikationen]

Mathematik-Online-Aufgabensammlung: Lösung zu

Aufgabe 1345: Gaußsches Fehlerintegral


A B C D E F G H I J K L M N O P Q R S T U V W X Y Z

Für $ a,\, \rho \,\geq\, 0$ seien

$\displaystyle K_\rho \; := \; \{ (x,y)^\mathrm{t} \in \mathbb{R}^2 \; \vert\; x^2 + y^2 \leq \rho^2 \mathrm{ mit } x,y \geq 0 \} \;,
$

und

$\displaystyle Q_a \; :=\; \{ (x,y)^\mathrm{t} \in \mathbb{R}^2 \; \vert\; 0 \leq x \leq a, \; 0 \leq y \leq a\} \; ,
$

definiert.

Zeige, daß die Gleichungen

$\displaystyle \displaystyle \int_{K_\rho} e^{-(x^2+y^2)} \, \mathrm{d}(x,y) = \dfrac{\pi}{4} \left( 1 - e^{-\rho^2} \right),
$

und

$\displaystyle \displaystyle \int_{Q_a} e^{-(x^2+y^2)} \, \mathrm{d}(x,y) = \left( \int_0^a e^{-x^2} \, \mathrm{d} x \right)^2
$

gelten.

Zeige damit die Identität des Gaußschen Fehlerintegrals

$\displaystyle \displaystyle \int_{-\infty}^\infty e^{-x^2} \, \mathrm{d} x = \sqrt{\pi}.
$


Verwenden wir die Polarkoordinatentransformation

$\displaystyle g(r,\varphi) \; =\; \begin{pmatrix}r \cos \varphi\\ r \sin \varphi \end{pmatrix} \; ,
$

so ist $ K_\rho = g(M_\rho)$ mit

$\displaystyle M_\rho \; :=\; \{ (r, \varphi)^\mathrm{t} \in \mathbb{R}^2 \; \vert\; 0 \leq r \leq \rho, 0 \leq \varphi \leq \pi/2 \} \; .
$

Mittels der Substitutionsregel errechnen wir

\begin{displaymath}
\begin{array}{rcl}
\displaystyle \int_{K_\rho} e^{-(x^2+y^2...
...= & \dfrac{\pi}{4} \left( 1 - e^{- \rho^2} \right).
\end{array}\end{displaymath}

Der Satz von Fubini liefert

\begin{displaymath}
\begin{array}{rcl}
\displaystyle \int_{Q_a} e^{-(x^2 + y^2)...
... \left( \int_0^a e^{-x^2} \, \mathrm{d}x \right)^2.
\end{array}\end{displaymath}

Es ist

$\displaystyle K_a \subseteq Q_a \subseteq K_{\sqrt{2} a}
$

für alle $ a \geq 0$ . Skizze für $ a = 1$ :
\includegraphics[width = 4cm]{s3.eps}

Da wir über eine nichtnegative Funktion integrieren, erhalten wir den Zusammenhang

$\displaystyle \displaystyle \dfrac{\pi}{4} \left( 1 - e^{- a^2} \right) \leq \l...
...2} \, \mathrm{d}x \right)^2
\leq \dfrac{\pi}{4} \left( 1 - e^{- 2 a^2} \right)
$

für alle $ a \geq 0$ . Im Grenzübergang $ a$ gegen $ \infty$ erhalten wir hieraus

$\displaystyle \displaystyle \dfrac{\pi}{4} \leq \left( \int_0^\infty e^{-x^2} \, \mathrm{d}x \right)^2 \leq \dfrac{\pi}{4},
$

d.h.

$\displaystyle \displaystyle \left( \int_0^\infty e^{-x^2} \, \mathrm{d}x \right)^2 = \dfrac{\pi}{4},
$

und somit

$\displaystyle \displaystyle \int_0^\infty e^{-x^2} \, \mathrm{d}x = \dfrac{\sqrt{\pi}}{2}.
$

Nutzen wir nun die Symmetrie der Funktion $ f:\mathbb{R}\to\mathbb{R}, \, x \mapsto e^{-x^2}$ bezüglich der $ y$ -Achse aus, so erhalten wir die Behauptung,

$\displaystyle \displaystyle \int_{-\infty}^\infty e^{-x^2} \, \mathrm{d}x = \sqrt{\pi}.
$

(Autoren: Künzer/Martin/Tentler/Wahrheit)

[Zurück zur Aufgabe]

  automatisch erstellt am 11.  8. 2006